Difference between revisions of "2007 AMC 8 Problems/Problem 15"

(Solution)
Line 20: Line 20:
 
==See Also==
 
==See Also==
 
{{AMC8 box|year=2007|num-b=14|num-a=16}}
 
{{AMC8 box|year=2007|num-b=14|num-a=16}}
 +
{{MAA Notice}}

Revision as of 01:24, 5 July 2013

Problem

Let $a, b$ and $c$ be numbers with $0 < a < b < c$. Which of the following is impossible?

$\mathrm{(A)} \ a + c < b  \qquad \mathrm{(B)} \ a * b < c \qquad \mathrm{(C)} \ a + b < c \qquad \mathrm{(D)} \ a * c < b \qquad \mathrm{(E)}\frac{b}{c} = a$

Solution

According to the given rules,

Every number needs to be positive.

Since $c$ is always greater than $b$,

adding a positive number ($a$) to $c$ will always make it greater than $b$.

Therefore, the answer is $\boxed{\textbf{(A)}\ a+c<b}$

See Also

2007 AMC 8 (ProblemsAnswer KeyResources)
Preceded by
Problem 14
Followed by
Problem 16
1 2 3 4 5 6 7 8 9 10 11 12 13 14 15 16 17 18 19 20 21 22 23 24 25
All AJHSME/AMC 8 Problems and Solutions

The problems on this page are copyrighted by the Mathematical Association of America's American Mathematics Competitions. AMC logo.png